opulence2001
Thanks Received: 4
Forum Guests
 
Posts: 43
Joined: November 10th, 2010
 
 
 

PT 20 S3 Q14: If an orange bead is the fourth...

by opulence2001 Mon Nov 15, 2010 7:43 pm

Hello,

For this question I eliminated all except D and E...and chose the wrong answer E.

I don't understand why E is incorrect if one has _ _ _ O (O is the fourth bead. Why couldn't we have _ Y P O.

R would have to go in position 1 which is blank, and positions 5 thru 8 could be an assortment of colours that could still meet the conditions like RYPOGOGG for example...

Any help is very much appreciated.
User avatar
 
bbirdwell
Thanks Received: 864
Atticus Finch
Atticus Finch
 
Posts: 803
Joined: April 16th, 2009
 
 
 

Re: PT 20 S3 Q14: If an orange bead is the fourth...

by bbirdwell Wed Nov 17, 2010 11:29 am

I don't understand why E is incorrect if one has _ _ _ O (O is the fourth bead. Why couldn't we have _ Y P O.


You cannot have that because of the very first constraint. I saw another post you made regarding #17, and you made the same mistake -- perhaps you looked over that constraint or misinterpreted it?

The first constraint says that anytime you have YP or PY, you must put an R on BOTH sides: R Y P R or R P Y R.
I host free online workshop/Q&A sessions called Zen and the Art of LSAT. You can find upcoming dates here: http://www.manhattanlsat.com/zen-and-the-art.cfm
 
dababbott
Thanks Received: 1
Vinny Gambini
Vinny Gambini
 
Posts: 7
Joined: March 10th, 2011
 
 
 

Re: Diagram

by dababbott Thu May 26, 2011 3:55 pm

Is this a game where a main diagram should be constructed? Doing this as an experimental section, I just saw the atypical game type, saw that all the questions were conditional, and dove into the questions.
User avatar
 
bbirdwell
Thanks Received: 864
Atticus Finch
Atticus Finch
 
Posts: 803
Joined: April 16th, 2009
 
 
 

Re: Diagram

by bbirdwell Sat May 28, 2011 4:54 am

Well, i've never worked a game that I didn't diagram. Doesn't mean you have to :)
I host free online workshop/Q&A sessions called Zen and the Art of LSAT. You can find upcoming dates here: http://www.manhattanlsat.com/zen-and-the-art.cfm
 
patrice.antoine
Thanks Received: 35
Atticus Finch
Atticus Finch
 
Posts: 111
Joined: November 02nd, 2010
 
 
 

Q14

by patrice.antoine Thu Feb 14, 2013 8:45 pm

Sorry, but this question is not in reference to Q14 but in reference to rule #1.

I totally despise this game because of rule #1. Doesn't "adjacent" mean next to in either order (YP or PY) ? If so, how do conditional questions like #16 and #17 NOT violate this rule???

And then for question #18, placing "PY" in the fifth and sixth rows infers that R must be in the fourth and seventh row for us to get to the correct answer choice.

I just do not understand how "PY" means we place R before and after that block in one question, but we don't do so in the other questions (question #13 and #17, for example). The correct AC for #13 has "PY"with no "R" before it and in #17 it has the same "PY" without requiring R to be before and after each. SO FRUSTRATING!!! :evil: :evil: :evil:

The only way I can see Rule #1 working is if it meant "unless purple or yellow are the first two beads, any bead that immediately precedes the pair must be red".

Please, can anyone help rectify this blatant LSAT contradiction!?!
User avatar
 
rinagoldfield
Thanks Received: 309
Atticus Finch
Atticus Finch
 
Posts: 390
Joined: December 13th, 2011
 
 
 

Re: Diagram

by rinagoldfield Sun Feb 17, 2013 8:58 am

Hey Patrice:

I also address this on your question about Q13, but here’s how we should think about rule 1:

The first rule says: If a purple bead is adjacent (next to) to a yellow bead, any bead that immediately follows and any bead that immediately precedes the pair must be red.

This rule does not mean that any PY pair must be bookended by two red beads. It means that IF a bead precedes the PY pair, then that bead must be read. Similarly, IF a bead follows the PY pair, then that bead must be red.

So:
X(PY) --> R(PY)
(PY)X --> (PY)R

patrice.antoine Wrote:The only way I can see Rule #1 working is if it meant "unless purple or yellow are the first two beads, any bead that immediately precedes the pair must be red".


Yup. Add to that "unless p/y are the last two beads, any bead that immediately follows must be red."

16 and 17 therefore don’t violate the rule.